Given that p is a prime? (Review/verify this proof)?

  • Thread starter Thread starter Math100
  • Start date Start date
  • Tags Tags
    Prime Proof
Click For Summary
SUMMARY

This discussion provides a proof that if p is a prime number and p divides a^n, then p^n also divides a^n. The proof begins by leveraging the definition of a prime, which states that if a prime divides a product, it must divide at least one of the factors. The conclusion is reached by expressing a as a product of primes and demonstrating that p must divide a, leading to the result that p^n divides a^n.

PREREQUISITES
  • Understanding of prime numbers and their properties
  • Familiarity with basic algebraic manipulation
  • Knowledge of divisibility rules in number theory
  • Experience with mathematical proofs and logical reasoning
NEXT STEPS
  • Study the Fundamental Theorem of Arithmetic for insights on prime factorization
  • Explore the concept of divisibility in number theory
  • Learn about mathematical induction as a proof technique
  • Investigate the properties of prime numbers in modular arithmetic
USEFUL FOR

Mathematicians, students studying number theory, educators teaching algebraic concepts, and anyone interested in understanding the properties of prime numbers and their applications in proofs.

Math100
Messages
817
Reaction score
229
Homework Statement
Given that ##p## is a prime and ##p\mid a^n ##, prove that ## p^n \mid a^n ##.
Relevant Equations
None.
Proof:

Suppose that p is a prime and ##p \mid a^n ##.
Note that a prime number is a number that has only two factors,
1 and the number itself.
Then we have (p*1)##\mid##a*## a^{(n-1)} ##.
Thus p##\mid##a, which implies that pk=a for some k##\in\mathbb{Z}##.
Now we have ## a^n ##=## (pk)^n ##
=## p^n k^n ##.
This means ##p^n \mid a^n ##.
Therefore, given that p is a prime and ##p \mid a^n ##,
we have proven that ##p^n \mid a^n ##.
 
Last edited by a moderator:
Physics news on Phys.org
You don't have to encapsulate every single letter or symbol. ## stands for "begin math mode" and "end math mode", so it is ##a^n = (pk)^n ## to get ##a^n=(pk)^n## or ##p \mid a^n ## to get ##p \mid a^n. ##

Here is your corrected code:
Code:
Proof:        Suppose that ##p## is a prime and ##p\mid a^n ##.
                 Note that a prime number is a number that has only two factors,
                 ##1## and the number itself.
                 Then we have ##(p*1)\mid a* a^{n-1} ##.
                 Thus ##p\mid a##, which implies that ## pk=a## for some ##k \in\mathbb{Z}##.
                 Now we have ## a^n = (pk)^n = p^nk^n ##.
                 This means ## p^n \mid a^n ##.
                 Therefore, given that p is a prime and ##p\mid a^n ##,
                 we have proved that ## p^n \mid  a^n ##.
 
Last edited:
Math100 said:
Homework Statement:: Given that ##p## is a prime and ##p\mid a^n ##, prove that ## p^n \mid a^n ##.
Relevant Equations:: None.

Proof: Suppose that ##p## is a prime and ##p\mid a^n ##.
Note that a prime number is a number that has only two factors,
##1## and the number itself.
Then we have ##(p*1)\mid a* a^{n-1} ##

We have ##p\,|\,a^n.##

The easiest way to continue is to use the definition of a prime: If it divides a product, then it already divides a factor. This gives us directly ##p\,|\,a.## Etc.

Another way is to write ##a=p_1\cdot \ldots \cdot p_m## as a product of primes. Then ##p\,|\,p_1^n\cdot\ldots\cdot p_m^n## and ##p=p_1## without loss of generality. Etc.
Math100 said:
Thus ##p\mid a##,...

Why? It is true, but why? You cannot conclude ##p\,|\,a## from ##p\cdot 1= a\cdot a^{n-1}## without explanation. E.g. ##4\cdot 1 \,|\, 6\cdot 6^2 ## but ##4\nmid 6.##

Math100 said:
... which implies that ## pk=a## for some ##k \in\mathbb{Z}##.
Now we have ## a^n = (pk)^n = p^nk^n ##.
This means ## p^n \mid a^n ##.
Therefore, given that p is a prime and ##p\mid a^n ##,
we have proved that ## p^n \mid a^n ##.
 
I'll use/apply the first way. Here's my revised proof:

Suppose that ##p## is a prime and ##p\mid a^n ##.
Note that if a prime number divides a product of integers,
then it must divide one of the factors from a product of integers.
This gives us ##p\mid a##, which implies that ## pk=a## for some ##k \in\mathbb{Z}##.
Then we have ## a^n = (pk)^n= p^nk^n ##.
Thus ## p^n \mid a^n ##.
Therefore, given that p is a prime and ##p\mid a^n ##,
we have prove that ##p^n\mid a^n ##.
 
Thank you!
 
Question: A clock's minute hand has length 4 and its hour hand has length 3. What is the distance between the tips at the moment when it is increasing most rapidly?(Putnam Exam Question) Answer: Making assumption that both the hands moves at constant angular velocities, the answer is ## \sqrt{7} .## But don't you think this assumption is somewhat doubtful and wrong?

Similar threads

Replies
5
Views
2K
  • · Replies 9 ·
Replies
9
Views
2K
  • · Replies 13 ·
Replies
13
Views
4K
  • · Replies 2 ·
Replies
2
Views
3K
Replies
30
Views
3K
  • · Replies 2 ·
Replies
2
Views
4K
Replies
6
Views
3K
  • · Replies 15 ·
Replies
15
Views
4K
  • · Replies 12 ·
Replies
12
Views
5K
Replies
2
Views
1K